Jump to content
BrainDen.com - Brain Teasers
  • 0


Guest
 Share

Question

Recommended Posts

  • 0

I believe the answer is A, because you duplicate the puzzle and put it edge to edge the squares follow a pattern. if you build one unified puzzle with the numbers in it for the dark squares. The numbers follow the pattern 5,1,1,5,11,5,11,5

post-41314-076368000 1298668236.png

I believe the answer is A, because you duplicate the puzzle and put it edge to edge the squares follow a pattern.

Link to comment
Share on other sites

  • 0

grids.jpg

Assigning values 1-9 (starting at the top and reading left to right as in the labeled square above), take the sum of the values of each column from bottom to top mod 10.

So for column 1: A1 + B1 = C1 (mod 10) or 7 + 1 = 8 and 8 mod(10) = 8; column 2: 8 + 7 = 15 and 15 mod(10) = 5

Then ? + 2 = 1 so ? = 9 giving i as an answer.

Link to comment
Share on other sites

  • 0

This is for the original puzzle...

First, given that the eight 3x3 grids can be numbered left-to-right and top-to-bottom as 1 through 8 with the missing item being number 9, and given that each 3x3 grid can similarly have its 9 squares numbered so that the top-left is "1" and the bottom-right is "9"...

Assume that the top row of grids are to be rotated 90 degrees clockwise, the middle row are not rotated, and the bottom row are rotated 90 degrees counter-clockwise. The top row grids' black spots would then occupy positions 4, 5, and 3. The middle row grids' blacks spots occupy positions 1, 7, and 2. The bottom row grids' blacks spots would occupy positions 9 and 6, leaving position 8 to be filled. To fill position 8 after a CCW rotation, we need grid "D" from the possible answers.

My answer is "D".

I tried a number of other revolving algorithms but they always resulted in at least one duplicate.

And if this is correct, I owe it to those who answered before me. I did not see this approach to a solution on my first three reads of the OP.

Link to comment
Share on other sites

  • 0

grids.jpg

Assigning values 1-9 (starting at the top and reading left to right as in the labeled square above), take the sum of the values of each column from bottom to top mod 10.

So for column 1: A1 + B1 = C1 (mod 10) or 7 + 1 = 8 and 8 mod(10) = 8; column 2: 8 + 7 = 15 and 15 mod(10) = 5

Then ? + 2 = 1 so ? = 9 giving i as an answer.

plainglazed, that was deep. I don't know if it's correct, but either way, nice work.

Link to comment
Share on other sites

  • 0

i think it's F

if you assign a numerical value to each square 1, 2, 3 (top row), 4, 5, 6 (middle row) and so on, and then make a chart of where the black squares are, you'll notice the following mathematical pattern:

the SUMS of each row and column (when added together with its "brother" is 30)

for example Row 1 + Column 1 = 30

Row 2 + Column 2 = 30

So, using algebra or trial and error, etc, you can figure out that the missing value for the 9th square must be SIX in order for row 3 and column 3 to add up to 30

here's how it looks, the numbers represent where the black squares are:

8 5 1 = 14

1 7 2 = 10

7 8 X = ??

16 * 20 * ??

16 + 14 = 30 (column 1 total + row 1 total)

20 + 10 = 30 (column 2 total + row 2 total)

(3 + X) + (15 + X) = 30

X = 6

so, pattern 6 is Row 2, Column 3 = F Pattern

get there....

Reasoning?

Link to comment
Share on other sites

  • 0

i think it is D because if we merge all blocks all smaller blocks in first row blocks come 2 times

in third row blocks come 2 times

and if we select option 'D', middle row blocks we come once.

so a pattern is being formed if we select option 'D'.

Link to comment
Share on other sites

  • 0

Makes sense, but wouldn't a simpler solution be assigning it 1-9, but starting from the bottom left square? Bottom left square is 1 and top right square is 9. So the first row it's 2+5=7, 2nd row it's 1+7=8 and last row is simply 1+2=3.

grids.jpg

Assigning values 1-9 (starting at the top and reading left to right as in the labeled square above), take the sum of the values of each column from bottom to top mod 10.

So for column 1: A1 + B1 = C1 (mod 10) or 7 + 1 = 8 and 8 mod(10) = 8; column 2: 8 + 7 = 15 and 15 mod(10) = 5

Then ? + 2 = 1 so ? = 9 giving i as an answer.

Link to comment
Share on other sites

  • 0

Makes sense, but wouldn't a simpler solution be assigning it 1-9, but starting from the bottom left square? Bottom left square is 1 and top right square is 9. So the first row it's 2+5=7, 2nd row it's 1+7=8 and last row is simply 1+2=3.

Frederico1, your solution is novel and valid, but I would not disagree with plainglazed's solution. It is also a novel and valid solution, even if it is a more complex approach. Lacking any clear direction from FoodForThought up to this point, both are reasonable and logical.

While I appreciate your solution I think that in the final analysis it will not be chosen as "correct" by FoodForThought because your approach to the "numbering" of the squares in the grid is counter to the ordinal positions ascribed by the originator of the puzzles - notice which solution is labeled "A" and which is labeled "I". This is just opinion, but I think that a person who would start at "A" at the top-left would also start at "1" for that position.

Link to comment
Share on other sites

  • 0

'E'

because first thing i noticed was that all the shades are among 'a','b','e','g' and 'h' so the answer must be one of them and second thing if you see you will notice that in a single row or column a shade is not repeating its place so in the 3rd column chances of 'a' and 'b' are over and in the 3rd row 'g' and 'h' are canceled so the remaining 'e' must be the answer

^_^

The shades in the 2nd row overlaps the shades in the first row at only 1 point and similarly the shades in the 3rd row overlaps the shades in the 2nd row at only one point

SORRY if i am not able to explain but you can see at the attachment:blush:

post-40192-091367800 1298793877.png

Edited by Shivam
Link to comment
Share on other sites

  • 0

1st Puzzle. I agree with Fredrico1, the answer must be... ]I. If the squares are assigned numerical values as follows:

789

456

123

- then we get 3 simple addition sums.

Link to comment
Share on other sites

  • 0

In my opinion, the answer to the first puzzle is B.

Reasoning is simple: All pieces need to have a symmetrical counter piece ( the symmetri axe is the horizontal middle row ).

You got 3 pieces that repeat 2 times. if you check it 2 of those pieces respect the symmetry line. the other piece that is double has just 1 symmetrical counter part, and that is the B answer.

The last piece, the one with the DOT in the middle, is it's own symmetrical counterpart , and for the sake of logic in that picture, the one piece that is missing is B

Link to comment
Share on other sites

  • 0

to the 1st puzzle, could it be that if we were to number the box, we will arrive at (8 5 1) , (1 7 2) and ( 7 8 x)

we then take the leap of faith that the connection between the 3 digits are a decreasing sum. 8 + 7 = 15, 15 + 6 = 21

we then eliminate the ten digits from each sum. leaving behind 8 5 1

for row 2, we have 1 + 6 = 7, 7 + 5 = 12

as for the last row, we get 7 + 9 = 18, 18 + 8 = 24

and thats how i arrived at option D

Link to comment
Share on other sites

Join the conversation

You can post now and register later. If you have an account, sign in now to post with your account.

Guest
Answer this question...

×   Pasted as rich text.   Paste as plain text instead

  Only 75 emoji are allowed.

×   Your link has been automatically embedded.   Display as a link instead

×   Your previous content has been restored.   Clear editor

×   You cannot paste images directly. Upload or insert images from URL.

Loading...
 Share

  • Recently Browsing   0 members

    • No registered users viewing this page.
×
×
  • Create New...